You are on page 1of 50

NY Bar-Video 1- Tort Law

 Tort Law in in both the Multiple Choice and the Essay


 Torts is not a statutory filed, it is a common law field

The 6 Main Topics/Modules that are going to be discussed are:

1. Intentional Torts & Defences


 There are 7 Intentional Torts
 7-8 multiple choice questions on Intentional Torts
(1) Battery
(2) Assault
(3) False Imprisonment
(4) Intentional Infliction of Emotional Distress
(5) Trespass to Land
(6) Trespass to Chattels
(7) Conversion
 There are 3 Affirmative Defences in Intentional Torts
1) Consent
2) Self-defence, defence of others, defence of property
3) Necessity

2. Duty & Breach Elements of the Law of Negligence


1) Prima Facie Case of Negligence
2) Standard of Care
3) Trespasses to Land, Licensees, Invitees,
4) Negligence Per Se (statutory standard of care)

3. Causation, Damages & Defences of the Law of Negligence

4. Strict Liability Claims including Strict Liability for Defective


Products

5. The Law of Nuisance

6. Miscellaneous Topics- Vicarious Liability etc.

7. Defamation and Privacy

Module 1: Intentional Torts and Defences


1
3 Overview Observations that Apply to Every Intentional Torts on the Bar Exam:
 1) When dealing with such claims the hyper sensitivity of the plaintiff should be ignored
in deciding if the plaintiff has a claim. Always assume you are dealing with a normal or
ordinary person, ignore eccentricities (oddness, weirdness, quirkiness, habit)

 2) In the law of Intentional Torts, there are no incapacity defences. People that lack legal
capacity elsewhere in the legal system, will be held liable for intentional torts. People
who lack capacity in the legal system are children, people who are intoxicated, under the
influence of drugs, suffering from a mental illness or disorder, developmentally disabled
or anyone who might be able to assert incapacity in another branch of the law is going to
be liable in intentional torts
 Example 1: if the 9-year-old locks you in a closet the child is liable
to false imprisonment
 Example 2: If the 8-year-old kicks you in the legal they are liable
for battery
 Example 3: The defendant is not liable because he lacked legal
capacity is ALWAYS A WRONG ANSWER and don’t
pick that choice
 3) In the law of Intentional Torts, if a defendant starts out with the intent of committing 1
tort on a particular person and a different tort results or a different victim is affected, the
defendant is liable for whatever happened. This is called the Transfer of Intent Doctrine
o You start at with the desire to commit Tort A against person X but you wind up
inflicting Tort B on person Y, you are liable for Tort B and for person Y
 Transfer of Intent Doctrine- It looks like an accident but he is still liable even though
he didn’t intent to hurt that person

1) Battery:
 There are 2 Elements to a Battery and the plaintiff must show:

 1) The Defendant Committed a Harmful or Offensive Contact


oOffensive Contact = unpermitted to a person with ordinary sensitivity
because hyper sensitive is not taken into account
o Offensive could be mental or emotional or physical act
o Harmful is a physical injury, if it makes you bleed, breaks a bone,
send you to the hospital, results in your death
 Example 1: P was studying in the library, D a big veracity
football player sees P and tells her ”I he really
like you” he then strokes her hair. She brings a
battery claim stating it was offensive contact
 Was D act to P offensive to an ordinary person?
o Yes, it was sexual harassment and unacceptable which constitutes an
offensive Battery. Going around touch people’s hair is an offensive
and will constitute a batter even though it’s not a sexual harassment.

2
 2) With the Plaintiff Person (extended personality rule) (with another's person without
that person's consent)
oYour person includes anything your holding, touching, anything you
are connected to
 Example 2: If P is walking down the street caring a purse
and D grabs the purse, it is considered a battery.
 Example 3: P went hors back riding and lost her way, she
sees a pedestrian and says “excuse me I lost my
way, do you know where the stables are
located?” The pedestrian then turns around and
slaps the horse on the ass.
 Does P have a cause of action for Battery?
o Yes, it was an offensive touching to her person (the horse). The horse
is part of her body as she was mounted on top of the hoarse.
 Example 4: P was at work when a new person tapped her on
the shoulder and said “excuse me do you know
where the bathrooms are?” Since childhood P
has had a phobia of people tapping her on the
shoulder and then has a reaction to it.
 Can P recover in an action suit for Battery?
o No, she will not recover, because that is not offensive to a person of
ordinary sensitivity, as it is permitted to normal people
o Eccentricities is not taken into consideration

2) Assault:
 There are 2 Elements to an Assault:
 1) The Defendant Must Place the Plaintiff in a Reasonable Apprehension
o Apprehension means knowledge

 There are 2 Tests in Apprehension:


 1) You go to see the defendant about to do something to you (see it
coming)

 2) You don’t have to be afraid


 Example 1: Justin Bebier picks up a brick and tells Dwayne
Johnson (The Rock) that he is going to beat him,
the Rock is not scared of Justine but there is
apprehension as he sees it coming and this does
constitute an Assault because you don’t have to
be scared, as all you need is knowledge,
awareness or perception that the battery might
soon be committed.

 The unloaded gun problem (the empty threat)

3
o If the defendant says he is going to commit a battery but lacks the capacity
to actually do it
 Example 2: D points a gun at P and says” Yo I am going to
shoot you” but the gun is not loaded
o As apprehension means knowledge, so if you know that the gun is loaded
then you know could be a victim of a battery. Therefore, it’s an Assault
o If you know the gun is unloaded, then you know you cannot be a victim of a
battery and therefore it is not an assault.
o The testable point is if you have no information, however reasonable
apprehension/reasonable relief is all that is required to establish an assault.
 An apparent ability creates a reasonable apprehension-
o It is reasonable to believe that a gun held at you by an assailant is loaded, as
people don’t usually carry firearms unloaded for not reason.
o If it’s reasonable to think he can do it, then it a reasonable that he can do it

 2) That Apprehension must be of an Immediate Battery


 2 Issues on the timeline of Immediate:

 1) Words alone (mere words) are not sufficient to create immediacy, there
must be conduct (conduct is required) :
 Example 1: D has his hands in his pocket and walks up to P
without making any gestures and tells her “in 20
seconds I am going to punch you in the face as
hard as I can and it’s going to hurt, I was a
professional boxer, 15 seconds clock it ticking”
 Does this place you in a reasonable apprehension of immediate battery?
o No because there was no placement of a reasonable apprehension of
immediate battery because words alone are not immediate. Even though
he said 20 seconds, talk is cheap and until you make a move its not
immediate.
 What kind of move and what kind of conduct?
o Menacing gesture it is often the display of a weapon the defendant takes
out a gun, knife or picks up a stick or a rock or it could be drawing your
hand back to slap someone

 2) Accompanying Words May Destroy Immediacy


o Sometimes you can have conduct and have the menacing gesture but
accompanying words may destroy/negate/gate(restrict) the immediacy
o On the exam if they give you both conduct and words combined, look at
the words and see what their natural implication is

 2 Examples where words negate the implication of conduct:

 1) Conditional Words
 Example 1: D walks up to P raises his hand about to slap her
and says “ if you were not my best friend I
would slap you silly”
4
 Has D placed P in reasonable apprehension of an immediate battery?
o No, P has no reason to believe she is about to be slapped because his
words said “if you were not my best friend” indicating she is his best
friend and he won’t slap her because she is his best friend so the words
indicate that he is not going to slap her.

 2) Words in the Future Tense


 Example 2: D walks up to P raises his hand about to slap her
and says “ just you wait until tomorrow and I
am going to slap you silly”
 Has D placed P in reasonable apprehension of an immediate battery?
o No because I have explicitly calendared this battery for tomorrow and
that’s not immediate

Practice Question:
 Billie Jean and Bobby are playing tennis. Every time Billie Jean is getting ready to serve
Bobby heckles her. After this had gone on for a while. Billie Jean trotted across the court
rapidly and began swinging her racket wildly as she approached Bobby, in order to scare
him. As she neared Bobby she slipped and the racket struck Bobby forcefully on his
hand, breaking a bone. Billie Jean slipped because the owner of the tennis club had
carelessly failed to clean some foreign substance of the court
 Does Bobby Have a Valid Claim Against Billie Jean for the Tort of Battery? Why or
Why Not?
o Yes, Bobby would win this claim because of transferred intent. By trotting across
the court Billie Jean had the intent to put him in apprehension of a battery.
o Her goal was to intimidate him to scare him by threating to hurt him, she ended
up committing the tort different then she intended

3) False Imprisonment:
 There are 2 Elements to a False Imprisonment:
 1) The Defendant Must Commit an Act of Physical Restraint
o Act of restraint does not only mean a direct physical restraint or a barrier
 Example 1: If D is holding P by the arm, preventing P from
walking away that is a battery but also false
imprisonment as D is holding P in place
 Example 2: If D sees P in a room and locks the door so P can’t
exit, this too is an act of restraint

3 Editorial Observations That Deal with an Act of Restraint:


 1) Threats are sufficient to be a restraint
 Example 3: P is sitting in a room working and D walks in and says
”if you leave this room in the next 30 minutes I’ll
shoot you” D then reaches into his coat and shows a
gun that proves he has the ability to do this. D then
leaves the room with the door wide opened and
unlocked.

5
Has D Committed an Act of Restraint? Would you leave the room?
o P would be scared to leave the room within the next 30 minutes as
D will shoot her, therefore that act would classify as an act of
restraint. A genuine threat that would act in the mind of a
reasonable or normal person because hyper sensitivity is not taken
into account.
 A common real-world threat of false imprisonment is:
 “If you leave I will call the cops” is enough to cause people to stay in place
 Usually this is done by Store security guards and rent a cop.
 When Wal Mart Security guards suspect someone of stealing and they say if you
leave I will call the police will constitute a threat for false imprisonment

 2) A Failure to Act & Omission, can be the requisite Act of Restraint if:
o A failure to act, omission and non-action can be a requisite act of
restraint, if there is a pre-existing duty between these 2 people
 Example 4: Pamela would like to fly from New York to Los
Angeles. Pamela has a physical disability which
requires the use of a wheel chair. Everything runs
smoothly until she gets to Los Angeles when the cabin
crew does nothing to help Pamela out, leaving her
sitting in the plane unable to leave under her own
power. The cabin crew failed to summon a wheel
chair
 This is an Act of Restraint just as much as if they
had threatened her

 3) An Act of Restraint Only Counts if the Plaintiff is Aware of it or


harmed by it
 Example 5: “P” and “D” are roommates but are not particularly
good friends. One-night D comes home drunk and
locks P in her bedroom as he thinks its hilarious. 2
hours later D has sobered up and realized this is a bad
idea as P will be angry, so he unlocks the door. When
P wakes up in the morning and walks out of her room
unimpeded and no problem. D then confesses the
prank to P that he locked her in her room while she
was sleeping and she was furious .
 Can P sue D successfully for false imprisonment?
o No, she cannot because for the 2 hours the door was locked, P was unaware of it
o While D did commit an act of restraint it was ineffective as she was unaware of it

 2) Plaintiff Must be Confined in a Bounded Area

 1 Testable Principle in a Confined Bounded Area

6
 An area is not bounded (not locked in) if there is a reasonable means of escape
that the plaintiff can reasonably discover
 Example 6: P has confined D in a space but the space is not
perfectly sealed up, there is a way out. The way out
will be dangerous or disgusting or humiliating or
hidden, then it is not a reasonable means of escape
and therefore the area is considered bounded and a
false imprisonment claim.
 Example 7: A man was looking to purchase a home, he went to go
visit the home. The home owner is there and the
visitor asks to check out the basement. Home owner
says yes and visitor goes down to the basement. Home
owner closes and locks the only door to and from the
basement on the visitor. In this basement on the far
wall there is a large rat-infested sewer pipe that leads
out to the street
 Does the Visitor have a Successful Claim for False Imprisonment against the Owner
who Locked the Visitor in the Basement?
o Yes, it was a bounded area because the rat-infested sewer pipe is not a reasonable
means of escape
 Example 8: A man is working out at a gym and after his workout
he goes into the shower area. While he is in the shower
someone takes all his garments. The only exit of the
shower is into a public place which is surrounded by of
opposite sex people. They are confined in a bounded
area because they have no reasonable means of escape
because the only means of escape is to walk out naked
and that would be considered humiliating

4) Intentional Infliction of Emotional Distress


 This claim can be made out by showing that the defendant behaved recklessly, you don’t
need to show intent
 Out of the 7 Intentional torts the one that does not require intentional conduct is the one
that has intentional in its name
 It could be that the defendant deliberately wanted to mess with the plaintiff emotions
 It is also sufficient if the defendant acted with outer disregard of the plaintiff’s emotional
tranquility

 There are 2 Elements to Intention Infliction of Emotional Distress:


 1) The defendant must engage in outrageous conduct
o Outrageous Conduct is conduct that exceeds all bounds of decency
tolerated in a civilized society
o Mere insults are never outrageous- Even if you acted with the intent to
distress you

 Outrageous Conduct, these factors will not always guarantee


o Conduct that is continuous or repetitive in nature
7
 Example 9: P would go to the photo copy machine and a co-
worker says a cruel and disgusting sexual
behaviour/language towards P. P is emotionally
distressed and can’t concentrate on her work and she
don’t know what to do
 Can P win? This was an intentional infliction of emotional distress, that was
outrageous conduct that exceeds all bounds of decency tolerated in a civilized society
o It will be a hard case to win, as disgusting as it is, it is not enough. But maybe if
it’s a jury question

1) What if the co-worker was saying the cruel and disgusting sexual behaviour every
day for a month?
 If P had to encounter the co-worker every day at work because there is no other way to
get her work done and every day the co-worker would say cruel and disgusting sexual
behaviour/language towards P for an entire month
 P would for sure win now because it will clearly be outrageous

 Abuse claiming debt collection practices


 They use threats if you don’t pay we are going to take your car
 If you don’t pay we are going to ruin your credit
 If you don’t pay we are going to foreclose your house
 Multiple times a day, every day for an entire month will be outrageous

2) If the defendant is a common carrier or innkeeper?


 Common carriers are transportation companies (airlines)
 Innkeepers are hotels
 They are supposed to treat their guests and their passengers with kindness
 If they do something with the intent to distress a customer we will always tag them
with the label of outrageous
 Example 10: P is traveling and it getting late and she has not made a
hotel reservation. P then sees a motel with a sign
vacancy. She then request a room for the night and the
hotel clerk says I don’t think so, you are too ugly to
stay here.
 This is outrageous, it is not normally outrageous for
someone to make fun of your appearance, however if a
hotel keeper does it then it is outrageous

3) Plaintiff is a member of a fragile class of persons


 There are 3 Types of Fragile Classes:

1) Children
 If a professor goes up to a student and says “ you are a son of a B*#@! And I
hate you” You go home depressed and don’t come to class for a week.
 If the student tries to bring a claim for emotion distress, they would lose
because it’s only a mere insult and is not considered outrageous

8
 If a professor goes up to a 5-year-old and says the same thing, the kids will
start crying and pooping all over and this is considered outrageous because a
child is a member of a fragile class.

2) Elderly People
 A professor is walking down the street and sees 82-year-old women, the
professor says you are a son of a B*#@! . Her reaction is irrelevant the purpose
is that this is outrageous as an elderly person is considered a fragile class

3) Pregnant Women
 The Defendant committing the emotional infliction must know the women it
pregnant, she must be really pregnant

 If a defendant has advance information that the plaintiff has a particular


emotional sensitivity or weakness then targeting that emotional weakness is itself
outrageous
 Example 11: You are at lunch with a co-worker and he tells you
he has a phobia that he is deathly afraid of kittens.
The next day you go out and buy a basket full of
kittens and put it on his desk, he then comes and
sees the kittens and has a heart attack
 Can the co-worker be successful in a claim for intentional infliction of emotional
distress?
o Yes, you are liable because you exploited a piece of information that you had
about his eccentricities.
o This is an exception to the principle that hyper sensitivity doesn’t count
because if you know about it and use it as a button then this is outrageous

 2) The plaintiff must suffer severe emotional distress


1. No specific Evidence is required – no particular kind is mandated
such as you don’t have to prove you lost work or went to see a
psychiatrist. You can use some of the evidence but are not
required to. Plaintiff will not lose if this are not provided
2. Negate the Element to Mildly annoyed is the opposite of being
severely distress. If its mile then it’s not severe. They can also use
the term momentarily upset, briefly irritated.

5) Trespass to Land
 2 Elements to Trespass to Land:
 An Intentional Act by the defendant causes a physical invasion of the
plaintiff’s real property
 You need to enter the land and go on someone’s property, by walking or driving on
the land

9
 A defendant is not necessary required to know that he/she crossed the
boundary line although it is intentional for him to want to walk on that
particular land even though he didn’t know it belonged to another.
 The defendant has to get to the specific land on purpose but you don’t need to
know you crossed the boundary line. It is the defendant’s responsibility in
Tort Law to know where the boundaries are located because property owners
are not obligated to mark their property
 Example 1: Dough is alone on a day hike, hiking on a state
park in which he is allowed. He then sees a fork in
the trial and he bares right and after a mile he has
left the state park. Dough is now on a farmer’s land
by the name of Pete, there was no fence, sign or
gate that would have signed to him that he left the
park. Dough is liable for the Tort of Trespass to
Land, whether he knew he was on Pete’s land or
not. It is intentional because he got to that location
intentionally as he walked and meant to be there.
 Example 2: Daniel is walking down the street one day and has
a heath attack and falls on Pete’s front lawn. Daniel
is not liable for a Trespass as he lacks intent. He
physically invaded that land but he didn’t get there
on purpose
 Another way to enter someone’s land is too throw something on the plaintiff’s land
or project something on the property
 The thing you throw on the land has to be tangible, physical be able to touch
it and feel it
 If it is intangible like shining or playing loud music on purpose to drive your
neighbor crazy, it is not trespass to land but is a nuisance.
 Example 2: If the defendant throws a rock at the neighbor’s
window and breaks the window he has to pay for
the damages because he trespassed to land by
intentionally invading the land

 2) The defendant need not have intended to commit a trespass, only to do the
act of entering the plaintiffs land
 Is the requirements that this invasion must relate to land including the occupier’s
space in the air above and the soil below out to a reasonable distance
 An airplane flying above is not trespass to land because it is beyond a
reasonable distance
 Example 3: A little boy throws a ball in the air, it touches
nothing and lands on the public street on the far
side of your house. That little boy will be liable
for Trespass. He penetrated the neighbour’s
airspace with a ball that is tangible object.
6) Trespass to Chattels
 An intentional act by the defendant that causes an interference
 An intentional interference with the plaintiff’s personal property

10
 Chattels means all your stuff, everything you own except land and buildings
such as vehicles, furniture, clothing, currency in your wallet
o cars, bikes, boats, coach desk, bed, desk, coat.
 The tort of Trespass to Chattels & Conversion are the private civil money
damages remedies for vandalism and theft of the defendant’s personal
property
 If someone damages your car or steals your wallet and they have money you
can sue them in one of these claims.

There are 2 ways in which someone can interfere with your personal property:
1) They can deliberately damage it by bending it, breaking it, scratch it, crush it,
throw it in the fire
2) They can take it away from the defendant – by stealing it, robbing it, abscond
with it(escape with it)

 What is the different between Trespass to Chattels and Conversion?


 The difference between the 2 is the magnitude of the interference
o It is a subjective test of what is grievous harm and what is modest harm
o If the degree of the interference is significant/great then the appropriate
cause of action is conversion
o If the magnitude is slight the appropriate cause of action will be Trespass to
Chattels
o Remedies are different in Trespass to Chattels and Conversion

7) Conversion- (Similar to Trespass to Land)


 An intentional act by the defendant that causes a serious interference with the plaintiff’s
right of possession in a chattel
 A mistake about the ownership of the item will not get the defendant off the hook, the
defendant remains liable
 An intentional interference with the plaintiff’s personal property
 Conversion plaintiffs get a special remedy:
 If it is categorized as conversion then you are not limited to covering cost of
repair, instead you can recover the full market value of the item in question

 Example 1: Professor is upset with one of his colleagues. He


leaves the building & keys the hood of his
colleagues. His colleague then sues him.
 Is this a big harm of small harm and what is his cost of remedy?
o Small harm so it is Trespass to Chattels
o Cost of remedy is cost of repair, whatever the body shop charges to fix the key on
the car the professor has to pay the cost of repair.
 Example 2: Professor is upset with his colleague. He grabs a
sludge hammer & breaks the windshield & 2
body panels of his colleague’s car.
 Is this a big harm of small harm and what is his cost of remedy?
o It will be conversion as it is big harm. He will not pay for the cost of repair but
instead the market value of the car.

11
o The book value of the car could be $25,000 instead of paying $6000 for the cost
of repair. If it is really damaged you have to buy it.
o “you break it you buy it”
 Example 3: P walks away from his work station at the library
and then you return. He put on your laptop but it
won’t recognize his password. It pisses him off
and throws it at the wall. Someone comes up to
P and says “excuse me why did you throw my
laptop at the wall?” P then realize he went to
the wrong desk and it wasn’t his computer.
 This is conversion and you still have to pay for the cost of
a new laptop. He acted intentional the mistake of you
thinking the laptop does not matter you are still liable
Practice Question:
 Dusty was a crop duster who was hired by a farmer named Chester to spray his crops with
insecticide. Duty typically used a county map to identify customers land and, on the map,
Chester’s farmer was marked as parcel 148, but in the email, Chester sent to Dusty, Chester
flipped the numbers and wrote 184 instead. Parcel 184 belongs to Roscoe, an organic famer.
Dusty sprayed Roscoe’s land while Roscoe was away, and when Roscoe discovered that his
crops were now covered in chemicals the organic supermarkets he sells to refused to accept
them and so he destroyed them.

 If Roscoe sued Dusty for Trespass to Land, which of the following is the most likely
outcome?:
A. Roscoe will lose because Dusty did not commit a physical invasion of Roscoe’s land
B. Roscoe will lose because Dusty’s actions were due to Chester’s mistake
C. Roscoe will lose because it was his independent decision to destroy the crops
D. Roscoe will win 

 Roscoe will when this because we have a physical invasion of the land by entering the
property, it was air space and still counts
 This was done by an honest mistake or bad information, however a mistake of entering
into the property does not remove him from liability

 Although it was Roscoe’s decision to destroy the crops, he destroyed them because of the
harm that followed from the Trespass and so it is recoverable

Defenses to Intentional Torts


1) Consent
 Consent is a defense to all 7 intentional torts
 Consent may be either express or implied (apparent or implied by law)
 The plaintiff must have capacity to consent and the defendant must not exceed the
bounds of the consent
 Exceptions to consent- If the consent was obtained through fraud or duress, then the
consent is void
 Fraud can also be a failure to disclose

12
 All consent has a scope, if a defendant exceeds this scope of consent, they defendant
should he be held liable.
 Consent is not all or nothing
 A 12-year-old child cannot consent to sexual intercourse
 Someone with limited mental capacity can only consent to those things that they
are capable of understanding
 Children and those with mild disabilities are able to consent to certain things like
wrestling but not surgeries
 If someone as sexual intercourse and they have a sexual transmitted disease they
can sue for Battery not because there was no consent but because it was a failure
to disclose the STD, which is a form of fraud

There are 2 Different Kinds of Consent:


 1) Express Consent- is an outright grant/explicit/direct permission to the defendant. It
could be oral or it could be written

 2) Implied Consent- There are 2 Different Kinds of Implied Consent:

1) Custom and Usage- If a plaintiff goes to a place or engages in an activity, where


certain invasions are routine. It is then assumed that the
plaintiff is agreeable to the invasion such as playing contact
sports such as American football or basketball. Playing ice
hockey and you get sent to the penalty box does not count is
false imprisonment because you consented to the game.
 You get in the subway during rush hour, you also consent to the pushes and
shoves that are incident to being in a crowded subway at rush hour.
 If you make a driveway to get to your house you are giving people an implied
consent to walk on that driveway by a custom of the community, customarily
acceptable that if someone wants to talk to you they can come up your front
lawn
 It is an implied consent to walk into a supermarket if the door is unlocked and
open for business that’s why you are not a trespasser
 Example 1: If playing American football, you get knocked
down by a player that is not an actionable battery
because you consented to being knocked down
when you agree to play the game because it is a
customary part of the play of the game
 Example 2: If you are playing sports there is an implied
consent. However, if playing the game, you fall on
the hardwood and another player stomps on your
hand that is a battery.
 It is a battery because that exceeds the scope of
consent as it is not part of the game
 Example 3: A doctor goes in for surgery on your right leg and
he also performs the surgery on your left, that
exceeds the scope of consent and is a battery.

2) A defendant’s reasonable interpretation of the plaintiff’s objective conduct


 Body language consent
13
 How the person is or was behaving in the past several minutes
 Look at the surrounding circumstances and draw reasonable inferences and
then act accordingly
 Doctors don’t get sued as they don’t need to say what they are going to do
because by going there you are giving them an implied consent, unless they act
inappropriately
 Example 4: Couple P & D are on a date, P asks if D wants
to come in. P comes in and D puts on candles
and gives D wine. They were both talking but
then it became quite and D goes in for a kiss. P
then tries to bring a claim for Battery
 D’s defense was that he said it was a
reasonable interpretation of objective
conduct. They have seen each other
previously and they had a good evening, you
closed your eyes and inclined your head. This
would all be pointed to as a basis of
reasonableness.
 There must be a greater circumstance for sexual intercourse to be acceptable. If it
happened it would be date rape and is not reasonable.

 Someone with limited mental capacity can only consent to those things that they are
capable of understanding, however incapacity is no defense.
 Example 5: P walks into a bar to have a beer and there is
another person who is really drunk and comes
up to P and punches him in the face.
 Is the drunk liable for Battery?
o Yes, because incapacity is no defense
 Example 6: P walks into a bar to have a beer and there is
another person who is really drunk and comes
up to P and says “You think you are so tough go
on and hit me”
 Is P liable for Battery if he punches the drunk after he consented?
o Yes, because even though he is telling you to hit him he lacks the capacity to consent

 A disabled individual who has the mental capacity of a 12-year-old goes into a hospital
and asks to have cosmetic surgery, can he give a valid consent to perform the surgery?
o No, his limited mental capacity makes that impossible
 Example 7: Two 11-year-old boys could agree to wrestle
with each other. If in the course of the wrestling
match one puts the other in a headlock.
 Battery claim will fail because they both have consented
 Example 8: P goes to your doctors and strips down and then
the door opens and it’s not P’s doctor it is
someone she does not recognize. P then ask who
they are and they say “Dr. Jones who is
covering your original doctor”. He then asks
her “if it’s okay to press down on her abdominal

14
to figure out what’s wrong with her” and she
says “press down as hard as you like just figure
out what’s going on” As he pressing her
stomach the door opens and it’s her doctor Dr.
Smith he then says “ Jones take your hands off
the patient and take off the medical coat and go
back to the cafeteria where you work and stop
impersonating a doctor”
 P then Sues Jones for battery, will she be successful?
o Jones says he had express consent, however it was fraud.
o He got that consent by telling her a lie(fraud)
o P will when and the consent will be disregarded because he obtained the consent
using fraud and it is a battery

2) Protective Privileges of: Self-defense, Defense of Others, Defense


of Property
 The defendant must reasonably believe that a tort is being or about to be committed
against himself, a third person, or his property
 Only reasonable force may be used (proportionality)
 Katko V Briney Case- Farmers held a trap with a shot gun in a shed because they
were being harassed by people who were trespassing. A
burglar goes in there and triggers the shotgun trap and
gets injured and sues the farmer. Farmer said defense of
property and the courts said the farmers can’t defend
their property with that gun.

 Deadly force is permitted if reasonably believed to be necessary to prevent


serious bodily injury
 Deadly force is never permitted to defend only property
 The shopkeeper’s privilege permits the reasonable detention of someone the
shopkeeper reasonably believes has shoplifted goods

 Must limit response to the force necessary under the circumstance, if excessive force is
used then you go back to a posture of liability
 Someone is about to punch you, you can shove them away or grab their hand you can’t
reach into your bag and start stabbing them that would be an overreaction and not self-
defense
 Deadly force can be used if someone has a gun or trying to protect a third person but
not if you are only protecting property, can’t shoot trespassers unless there are other
circumstances
 They all involve a defendant responding to a threat that is emanating or coming from
the plaintiff.
 The plaintiff is threating the defendants:
 Physical safety,
 Physical safety of a third person,
 The defendant’s stuff and
 The defendant wants to take pro-active actions in order to mitigate or prevent this
threat for happening
15
 A protective privilege is only available if the treat is in progress or emanate, it has to
be happening right now or is about to happen or in the heat of the moment, in real time
 Timing can make the protective privilege unavailable: Acting to soon or acting to late
 Acting to soon- If someone says tomorrow they are going to slap you silly and you
slap them first, this is not self-defense you are not acting in the heat of the moments
and you acted too soon.
 To Late- Someone comes up to you and punches you in the face and walk away they
get to half a block down the street, you then run after them and knock them down to
the pavement. This is not self-defense because he acting too late. (No revenge)
 Accuracy- The defendant must have a reasonable belief that the threat is genuine, if he
makes a reasonable mistake he won’t be liable and still has the defense available
 Example1: D is returning from a business trip. D is waiting
to pick up her bag which has a colorful ribbon on
it, P goes and picks it up and starts walking out of
the terminal with it. D runs after P and grabs the
bag out of P’s hand. P says that my bag looks at
the label. D then notices it’s not her bag and
apologies. P then sues D for battery because when
P grabbed the bag D was holding it and P grabbed
it in a very abrasive way. D argues in return to
defense of property that she had a reasonable
belief that it was her bag and her argument will
prevail (win) as it was a reasonable mistake.
Practice Question
 Darlene, who was worried about a series of burglaries in her neighborhood, purchased a gun
for protecting. That evening, Pat, who was homeless and trying to get out of the cold,
entering her garbage through an unlocked door. Darlene hear some noise as Pat began
rummaging around for some blankets. She opened the door to the garage with her gun in
hand. Pointing it in Pat’s direction in an attempt to frighten Pat away, she shouted, “Get out
of ill shoot! “and fired what she thought was a warning shot into the ceiling. Unfortunately,
her aim was bad and a bullet struck the homeless person in the shoulder.
 If Pat sues Darlene for battery, what is her best defense?
A. Pat was a trespasser
B. Darlene used reasonable force to protect her property
C. Darlene did not intend to hit anyone
D. Darlene reasonable feared for her life
 The fact that someone is a trespasser does not give anyone the right to shoot them
 If she was protecting property she could not use reasonable force by using the gun
 Whether she intended to shoot/hit him does not matter as she did intend to scare him,
under transferred intent she can’t rely on the argument that she didn’t intend to hit him
 The best argument is that she feared for her life because when someone trespasses a
home, it is not just protecting property but your physical safety and you may make the
further assumption that they may be armed and deadly force to protect someone who has
invaded your home will be acceptable to most courts.

3) Necessity
 Necessity defenses only applies to the 3 claims in property torts such as: Trespass to Land,
Conversion, Trespass to Chattel. If not one of these 3 it will not work
16
 There are 2 Types of Necessity Defenses:
 1) A defendant whose property tort was justified by a public necessity has an
absolute defense
 If a defendant commits a property tort in an emergency to protect the
community as a whole or a significant group of people, if this is true it will be
an absolute complete defense and no liability.
 An emergency is usually a pretty big catastrophe, a public emergency
threatening significant harm
 Example 1: A fire/hurricane is spreading throughout a city; the
wind is spreading it from 1 building to the next. It
is getting close to a gasoline refinery where it will
explode and cause a great loss of life. Dave wants
to mitigate the harm and deal with the crisis as a
hero and has to go on someone’s property and use
someone stuff. Dave sees a store that sells fire
stuff and smashes a window to get equipment to
put out the fire successfully.
 The store sues Dave for 2 torts: Trespass to Land by breaking the window and going in
the property and Conversion because he took and used all those fire extinguishers. Is
Dave liable?
o Dave is not liabable because of public necessity

 2) If justified only by a private necessity, the defense is qualified (the


defendant must pay for any damage caused)
 Commits a property tort in an emergency but does so for personal reasons, to
protect the interest of his own not for the public.
 The catastrophe threatens his own safety or his own possessions, so then he
takes something that belongs to someone else or enters someone else’s land in
order to protect his stuff or his bodily security
 This is only a partial defense
 There are 3 Legal Consequences in a Private Necessity Scenario:

1) You have to pay for any actual harm you caused to the plaintiff property
(compensatory damages)

2) If you do no harm, your technical tort is excused and therefore you will
not be held liable for nominal or punitive damages

 This privilege trumps (wins) a property owner’s right to defend his property
3) If you go on someone’s property during the emergency to take refuge,
you must be allowed to remain. The property owner is not allowed to
throw you off the property as long as the emergency continues (a right of
sanctuary)

 Example 1: Dave is a country hiker and he in hiking in rural


Minnesota in February. A sudden fierce unexpected
blizzard arises and the weather drops to -40 where it
is intolerable. Dave realizes if he doesn’t take shelter

17
he is going to die. He then finds a farm house
owned by famer Pete. Dave bangs on the door of the
farm house and there is not answer,
 3 Different Scenario Endings:
 1) He breaks the window and climbs in and takes refuge in the farm house overnight.
When the storm was gone in the morning he gets up and is on his way. Farmer Pete has
security cameras and sues Dave for Trespass to Land and pay him for the broken
window.
 Famer Pete will win and Dave will be liable for the cost of the window
because it was only a private necessity and there was not public interest
involved, so he must pay for the harm that Dave did.

 2) The door is unlocked. Dave then walks in and takes refuge in the farmhouse overnight.
Dave doesn’t touch or break anything or even take food or beverage. When the storm was
gone in the morning he gets up and is on his way. Farmer Pete has security cameras and
sues Dave for Trespass to Land as he wants nominal damages, it’s the principle.
 Farmer Pete will lose that case as no technical trespass. In an emergency you
have the privilege to enter, if you have a privilege to enter it’s not a trespass
and you can’t recover if someone has a privilege
 He can’t get punitive damages either because without a technical tort there is
no basis for punitive damages

 3) The door is unlocked. Dave then walks in and takes refuge in the farmhouse overnight.
10 minutes after Dave arrives farmer Pete comes down the stairs and tells Dave to get out
of his house and shoves him out of the front door, pulls the door closed and locks it.
Farmer Pete will be liable for any injuries sustained to Dave from being thrown out in the
blizzard (frost bite, pneumonia, dies).
 You normally have the privilege to eject a trespasser but not during a
blizzard, you have to tolerate their presents

Module 2A: Negligence Duty and Breach


o 50% of MPT questions will be on Negligence
o Out of the 27 questions you should there will be 14 Negligence questions
o 7-8 on Intentional Torts Questions
o 4-5 questions on all the other topics

Tort Elements of the Prima Facie Case of Negligence


 There are 4 Elements to the Tort of Negligence:

 1) The defendant owes a duty of care to conform to a specific standard of


conduct
 The plaintiff must show that the defendant owed a duty and then specify the
nature of that duty
 You owe a duty of care to all foreseeable victims (people who are near you),
 You don’t owe a duty to unforeseeable victims (people who are far away).
 Zone of danger use common since if it’s a nuclear bomb it’s the whole
world if it’s chopping down a tree it has to be the height of the tree.
 Palsgraf V Long Island Railroad Co. Case
18
 Two men attempted to board the train, one of the men was aided by railroad
employees who then dropped a package that exploded and caused a large coin-
operated scale to hit Ms. Palsgraf causing her injury. Ms. Palsgraf sued the
railroad, arguing that its employees had been negligent while assisting the man,
and that she had been harmed by their negligence. Her claim failed because she
was not a foreseeable victim and no liability to unforeseeable victims.
 General rule is Judge Cardozo View (Majority)
o Foreseeable Zone of Danger:
 He said Ms. Palsgraf was not owed a duty of care because she was outside the
zone of danger
 People around/near you are the people who could get hurt
 Judge Andrews View (Minority)
o Everyone is Foreseeable:
 Defendant owes a duty of care to the world at large regardless of whether the
plaintiff was a foreseeable victim

 The Exceptions to the Palsgraf Case is


 A rescuer can be out of the zone of danger but comes to the center of the action to
give aid/comfort/assistance to someone who is more directly injured
 If the rescuers in the process sustains injury he will be able to sue in Negligence
 They are not subject to the zone of duty principle
 Wagner v. International Railway Co. Case
 Cardozo stated “Danger invites rescue” so we will treat these people
as foreseeable victims even if out of the zone of danger

 2) The defendant breached that duty


 The plaintiff has to show that the defendant failed to live up to the duty of care by
a specific way by an evidentiary showing

 3) The breach of duty was the actual and proximate cause of the plaintiff’s
injury
 Causation comes in 2 different tests such as factual cause and proximate cause

 4) The plaintiff suffered damages to person or property


 You have to show that something happened to you

Practice Question:
 The DeLux corporation owns and manages an apartment building in an upscale and
generally safe neighbourhood of a large city. There is a garage that is part of the apartment
building and a door that leads from the garage out to the street. The door is designed to be
opened only from the inside so you can exit out to the street but it is not supposed to be
opened from the street. However, they unreasonably failed to maintain the door. As a result,
the door was able to be opened from the street side. One evening Patricia, who does not live
in the building is walk down the street when she in confronted by a mugger. The mugger not
wanting to commit a crime in plain view on the street, tries the door and is able to open it.
He drags Patricia through the broken door and into the DeLux Apartment garage where he

19
assaults her, takes her purse and then flees. Patricia sues DeLux for negligence, citing their
poor maintenance of the garbage door.
 Is she likely to win? Why or why not?
o Patricia will lose because she is an unforeseeable victim and is not owed a duty of care
as she was just a passerby on the street.
o The foreseeable victims would be the residents of the apartment building, who might
sustain criminal attacks once in the building

Standards of Care:???
 Subjective Standard of Care
 Must exercise the same degree of precaution as would have been exercised by a
hypothetical reasonable prudent person acting under similar circumstances
 Reasonably prudent person has no height, age, weight, sex, race and no physical
attributes
 A reasonably prudent person has a set of behaviors, they are always alert, careful and
always taking advance precaution. Most people to live up to this standard.

 Objective Standard of Care


 We make no allowances for the defendants particular short coming, everyone must
live up to the standard of an objective standard
 Inflexible, ridged and harsh standard
 Example 1: Pete claims that Dave kept gasoline-soaked rags in
his unventilated garage during the hottest months of
the summer. The rags caught fire and burned Dave’s
own garage and the fire spread next door and burned
Pete’s garage as well. They are both detached
garages. No personal injury or damage to the houses
 Pete sues Dave in a Negligence claim for money to the burned garage, Dave tells the
court he is stupid and below IQ levels. Is Dave still liable?
o Dave is still liable because he has to do as good as the reasonable prudent person
and he didn’t
o If the defendant is mentally disabled and has the mental age of a child of 6-7 years
old, that person has to do as good as the reasonable prudent person
o Beginner who engages in an activity for the first time, still has to do as good as
the reasonable prudent person

2 Exceptions to the Reasonable Prudent Person Standard of Care:


 1) If a defendant has superior skill or knowledge
 The standard of care includes a superior skill or knowledge of a hypothetic person
with that same superior skill or knowledge
 Act like a reasonable prudent person with that extra knowledge, as he has superior
knowledge like a professional race car driver
 The Reasonably Prudent Person Standard of care will go higher for professional
knowledge but it will never go lower for the stupid clumsy people

20
 2) Where relevant defendants’ physical characteristics are taken into the standard
of care (average mental ability but the same physical characteristics as the
defendant)
 Where relevant defendants’ physical characteristics taken into the standard of care
 If the defendant is blind, the standard of care will be of a reasonably prudent blind
person, it is not expected for a blind person to see things as they don’t have vision
 We expect a reasonably prudent blind person to take precautions appropriate to
visual impairment, we expect them not to go out without a cane, guide animal or
some assistant device
 Physical characteristics are not always relevant
 Objective, default standard of care that applies in every case automatically by
operation of law

6 Special Standards on Negligence Claims:


 1) Children over the age of 5 must conform to standard of care of a child of like age,
education, intelligence, and experience of a child in similar circumstances
 Exceptions: If a child is engaging in an adult activity the reasonably prudent person
standard of care applies (Adult activity is operating a motorized vehicle)
 Children under the age of 5 owe a zero duty of care
 Every child is subject to a different standard of care, it is a subjective standard so
it varies from child to child and that makes it flexible and lenient
 Pro defendant standard of care, it is hard to win negligence cases against children
 For liability we look at a child of similar age and experience

 2) Professionals must exercise the knowledge and skill of a member of the profession
in good standing
 A standard of care of the average member of the same profession, providing
similar professional services
 Lawyers, accountants, architects, engineers, health care providers such as doctors,
dentists, chiropractors
 A plaintiff in a mal practice case always needs an expert witness, which can be
from any state and not only from the same state because how are we going to
know what is customarily done
 Doctors owe a duty to disclose risk of treatment to patients, prior to embarking
upon that treatment. (informed consent doctrine).
 If doctor withholds that information or fails to share it and if the risk materializes,
you will have a claim in negligence
 When getting a professional negligence questions don’t use the word reasonable it is
not reasonable it is average member

NY Bar - Video 2 -Tort Law


Module 2B: Negligence- Duty & Breach

3) Premises Liability Cases:


 Someone enters a piece of property and while they are on that piece of property they
encounter a dangerous condition

21
 The question will always be what duty of care does the person in possession of the
property owe to the entrant on that property
 The types of piece of property can be a piece of outdoor open land, a tract of land in a
forest that is being held for future development, a field/orchid owned by a farmer, or a
vacant lot in a city, or an outdoor developed parcel like an amusement park or building,
office building or a commercial establishment like a supermarket or a private home
 Ranges of hazards that someone may encounter entering property can be a chair in your
doctors waiting room, chandelier that is not properly bolted in the ceiling, Icey front steps
 The duty of care is different based on the kind of entrant we are dealing with

A Question on Premises Liability


 Step 1: If it’s a premises liability question, you ask did someone come on land?
 Step 2: What kind of entrance am I dealing with?
 Step 3: Then use the 4- or 2-part test?
 Step 4: Apply the test to the hazard that hurt the plaintiff

Practice Question
 Darryl owns a large ranch. Over a deep canyon there is a wooden footbridge. He recently
had it inspected by an engineer. The engineer told him that, even though it appeared safe,
the footbridge had rotted so that it could no longer support the weight of a person. One day
while out horseback riding he sees Pete, a hiker on his land without permission, walking on
his land towards the bridge.
 Does Darryl owe Pete any duty to protect him from getting hurt on the bridge? Why or
why not? (apply the facts to the 4-part test)
o There is a duty because there is a known trespasser, Pete did not enter with permission
but Darryl has firsthand awareness of his presence
o The duty that Darryl owes a duty to Pete to (4-part test) artificial, highly dangerous,
concealed and prior knowledge
Standard of Care to Landowners
 A landowner’s standard of care under traditional rules usually depends on the status of the
person injured on the property

1) Unknown Trespassers
 The landowner owes no duty to undiscovered trespassers
 An unknown trespasser always loses a negligence claim in a premise liability case,
you can’t win if you can’t establish a duty and no duty is owed so you will lose
 An unknown trespasser is an unforeseeable victim

2) Known Trespassers
For discovered and anticipated trespassers, the landowner owes a duty to warn of or
make safe known highly dangerous artificial conditions if not obvious to the
trespasser
 Anticipated trespassers are those in which you should expect to trespass even if you
don’t have actual and concreate awareness that they are on the property
 On the exam-they are signally anticipated trespassers when they tell you there has been a pattern
of trespassing in the past because if they people have done it in the past they will continue to do so in
the future, so you should expect them

22
The duty is that the possessor must protect only against those hazards on the land
that meet a 4-part test:

1) The Conditioning in Wuestion Must be Artificial in Order to Trigger a Duty


 Artificial means constructed by human beings and not natural
 A possessory owes a known of anticipated trespasser no duty with respect
to natural occurring conditions

2) The Condition Must be Highly Dangerous


 Highly dangerous means capable of inflicting severe bodily injury or death
 No duty to moderately dangerous conditions

3) The Condition Must be Concealed from the Trespasser


 The trespasser must have no advance knowledge and it has to be a hidden
condition
 No duty of care in respect to open and obvious conditions, as the trespasser
can see it he can take the necessary precautions he/she might wish to take

4) Possessory of the Property Must have Prior Knowledge that the Condition Exists
 If you don’t know about it you owe no duty

3) Licensees
 Licensees are those who come onto the land with express or implied permission/
consent but for their own purpose
 They do not confer any economic benefit on the possessory of the property but
they have either an express or implied permission
 Most common type is a social guest, a friend you invite over for dinner
 Someone that comes to your door like a politician seeking your vote or girl
scouts selling cookies, religious people trying to convert you

 The landowner’s duty is the same as for discovered trespassers except that it applies to all
known dangerous artificial and natural conditions
 Duty is Defined in a 2 Part Test, Only Need to Protect Those That Are:
1) Concealed from Them
2) Known in Advance by You

4) Invitees
 Invitees are those entering as members of the public or for a purpose connected to
the business of the landowner
 Invitees enter with permission and they enter to confer economic benefit on the
possessory
23
 You are also an invitee if you enter property that is open to everyone, it is open to
the general public
 A business customer is an invitee such as when you go to the supermarket, or hair
salon, or go to the airport to pick up a friend without doing any commerce
because the airport is open to the public at large or enter a museum that is open to
the public that does not charge in admission, or a house of worship

 The landowner’s duty is the same as for licensees but with the additional duty to
reasonably inspect for dangerous conditions

2 Part Test, The Duty Arises Only When the Condition is:

1) Concealed from the Invitee

2) Condition Must be Either Known by the Possessor or Could be


Discovered Through a Reasonable Inspection
 A reasonable inspection is conducted by a reasonably prudent person.
It is thorough and takes cost into account and occurs at reasonable
intervals, it does not require to be inspected every day or every hour
 There will still be some hazards that could not be discovered through a
thorough inspection because they are so well hidden, to those you owe
no duty

 Note that the ordinary reasonable care standard applies for active operations on the
property and for conditions on the land that injure children (the “attractive nuisance”
doctrine)

Premises Liability Scenarios


1) Firefighters and Police Officers
 Firefighters and police officers frequently come on land, it is in the nature of their
job to enter property
 They are never allowed to recover for injuries that are inherent risks of the job
because they have the theory of assumption of the risk as they know what they are
getting into
 What are the inherent risks of a firefighters?
o Getting burned, smoke inhalation, getting hit by falling pieces of drywall or
timber as you enter that burning building
o If this happens you don’t categorize him as a kind of entrances or go through a 2-
4-part test
 What are NOT Inherent Risks of Police Officers Jobs?
o They come to a house for a domestic dispute and sit on the chair and it collapses
o It has to be linked up with the work

2) Trespassing Kids (attractive nuisance doctrine)


 A property owner must exercise reasonable prudent care to prevent injuries to
trespassing children from artificial conditions on the land
 Assess the likely hood that children will come on the land
24
 If the land is in the middle of nowhere and a neighbour is 10 miles away, the odds of
trespassing children a not likely, it would be reasonable to leave hazards on the land
 If the land is across the street from a middle school and a swing set is visible in his
backyard, then assessing the likelihood of trespass is very high
 The swing set acts as a child magnet, an assessing the likelihood of trespassing children
ask yourself if there is anything any item that is visible in your land that will draw
children
 If there is an item that is a child magnet then reasonable prudent dictates that he should
make some effort to make the property safe

3) If we have a Duty that is Owed in Any Scenario Other than Trespassing Children
 If a duty that is owed to an adult, trespassing adult or licensee or invitee

 There are 2 ways in which you can satisfy your duty:

 1) Fix the Hazardous Conditions, to Eliminate the Danger


 To repair or make the condition safe
 If you have ice on your drive way put salt to protect guest from slipping
and falling on the snow
 If you have an unstable chair fix it or throw out that chair

 2) Give a Warning
 Instead of fixing the item itself you can provide information
 You can give either an oral warning or you can use a sign
 You can tell your guests don’t sit on that chair it’s kind of flimsy
 You can put up a sign do not cross this bridge, it is icy and put a chain
4) Statutory Standards of Care (Negligence- Per Se)
 In the law of Negligence plaintiffs will often search for a criminal Statute that covers the
conduct that the defendant engaged in and ask the judge to use that statue as a one time
this case standard of care, by shutting down and eliminating any possibility of excuse
 There is a criminal statute that say you can’t run a red light and if you do you will
be subject to a fine and points on your license
 If you find that he ran the red light, you must find that he was negligent

When Can You Use a Criminal Statute in Civil Litigation to Prove the
Standard of Care? (2-part test of class of person class of risk test)

 1) Is the Statute designed to protect a class of people to which this plaintiff


belongs to or is a member of

 2) Must prove that the statute is to prevent the type of risk that materialized in
this case. The injury that occurred is the kind the statute is trying to prevent

 If the test is satisfied then you can use the statute


 If you can prove a statutory violation, that statutory violation is conclusive of a breach
and no excuses are allowed
 This is called Negligence- Per Se

25
Practice Question:
 Delores goes to work unware of a slow gas leak from the stove in her kitchen. That evening,
when she returns to her apartment, she is very stressed, so to relax she decided she would like
to smoke some marijuana. When she lights the joint there is an explosive from the gas leak
which blows out the wall of the apartment and damages the furniture of Delores’s neighbor,
Patricia. Marijuana is illegal in this state, punishable by 3 months of jail time. Patricia sues
Delores for negligence.
 Can Patricia use the criminal statute regarding marijuana use as the standard of care?
Why or why not?
o Patricia will not be able to use this statute because she can’t establish the 2-part
prong of class of person class of risk
o The statute that criminalizes marijuana is designed to protect against health risks not
from blowing up your apartment
o The statute is designed to protect the marijuana user from her own decision of using
the drug, it is not designed to protect next door neighbors
 Therefore, it fails the 2-part test and you cannot use the statute but she can
use the prudent reasonable person standard of care

 What does a reasonably prudent person do when they walk into a house with a gas leak?
o They usually would smell it and call the fire department
o A reasonably prudent person would not smoke marijuana or use any fire when there
is a smell of gas

Exceptions to the Statutory Standards of Care on Negligence- Per Se


 1) Even when the 2-part test is met, if statutory compliance/follow would have been
more dangerous than the statutory violation, then don’t borrow the statute
 Example1: Dave is driving on a road that has a yellow line that means
you cannot pass to the other side. A child then runs onto the
road and Dave swerves over the yellow line to avoid hitting
the child. Pete a bicyclist sees Dave swerving in the other
lane and he the swerves and hits a tree. Pete is badly hurt
and sues Dave. Pete wants to borrow the statute that says
you do not cross a double yellow line. Class of person class
of risk is satisfied here, it’s a traffic ordinance protecting
motorists from head on collisions. Pete will not be able to
use the statute because if Dave didn’t go over the yellow
line he would have hit the child. Dave violated the statute in
order to mitigate or avoid much more serious harm. This
will be analyzed under a reasonable prudent person under
the circumstances.
 2) If statutory compliance was impossible, don’t use statute even if the 2-part test is
met
 Example2: Dave a motorist has a heart attack and hits a pedestrian.
Motorists runs a red light, so he will use the red-light statue
and class of person class of risk is satisfied. But if you have

26
a heart attack and can’t control your vehicle, statutory
compliance is impossible. Dave can’t stop the car because
he is incapacitated. Here we fall back on a reasonable
prudent person standard of care.
 Plaintiff wants to know if Dave had chest pains prior to the heart attack because if he was
a reasonable prudent person he would have pulled over to the side of the road and dial
911 on his cell phone or
 If he is supposed to be taking heart medication and missed it this morning, as this would
be an unreasonable thing to do
 The violation of the statute will not be the standard of care

5) Duties to Act Affirmatively (Special Standards/duty in the Law of Negligence):


 There are no duties to act affirmatively by taking a course of conduct, if you don’t want
to. If you don’t want to drive & take a bus then you don’t have to drive & take a bus, no
one will make you. However, once you decide to drive you have to drive as a reasonably
prudent person would, including taking affirmative steps to drive as a reasonably prudent
person
 You have no duty to act to rescue a person in peril, evening if a child is drowning

 Exceptions 2 Scenarios Where There is a Duty to Rescue:

 1) If there is a pre-existing relationship between plaintiff & defendant,


that triggers a duty
 Employer-employee, land owner-invitee, innkeeper-guest, common
career-passenger

 2) If the defendant is the one who caused the peril or put the plaintiff in
peril, that triggers a duty
 The duty in both of these scenarios, are a duty to act reasonably under these circumstances
 You are never obligated to put your own life in jeopardy and rescue may not be feasible
under the circumstances
 If you opt to rescue and you perform the rescue negligently, you are liable
 Many states changed that rule with a statute called good Samaritan laws

6. Negligent Infliction of Emotional Distress (Special Standards/Duty Situations)


 General basis of liability: The defendant breaches a duty to the plaintiff by creating a risk
of physical injury and the plaintiff suffers emotional distress as a result

There are 2 Requirements to Recover in a Near Miss Case:


 1) The plaintiff must be within the “zone of physical danger” and
 2) As a result of the emotional distress you then suffered subsequent physical
manifestations

Exceptions:
 The negligent defendant breaches a duty to a bystander not in the zone of danger who
 (I) Is closely related to the injured person,
 (ii) Was present at the scene of the injury, and
27
 (iii) Personally, observed or perceived the event

Business Relationship Cases


 Involve a plaintiff and a defendant who are in a pre-existing business relationship, where
careless performance is highly likely to cause emotional distress

 1. Medical Patient & Medical Laboratory


 Example 1: P goes to the doctor because she has a strange growth in her
arm. The doctor takes a sample and sends it to the lab. The
lab negligently performs their work and informs P that she
has cancer.
 Is P Emotionally Distress? Is this Foreseeable?
o Yes, she is emotionally distress, if you are a lab testing for cancer you know that
screwing up will have this consequence
o This is a good case for a business relationship of negligent infliction of
emotional distress

 2. Funeral Parlor & Patron


 Example 2: P goes to a funeral establishment, she is already emotionally
fragile. She is making arrangement for a family member.
 Is it foreseeable that if they screw up by cremating instead of a casket or they lost the
body, will P be more distress?
o Yes, this is a good claim for negligent infliction of emotional distress
 3. Dry Cleaner & Customer
 Example 3: P brings in her favourite shirt to the dry cleaner. 3 days later
she comes to pick up the shirt and the dry cleaner has
destroyed the shirt. She is screaming in emotional distress,
however in this case there is no business relationship of
negligent infliction of emotional distress.
 Is it not foreseeable that screwing up the dry cleaner will cause emotional distress?
 It is not a relationship predicated on that

Practice Question:
 Pamela was sexually harassed by her manager at work. She managed to surreptitiously
record some video of the episode on her cell phone. The next day she asked a co-worker
named Carl to held her file a complaint with the EEOC, and she shared the video with
him to show what had happened. Carl helps her file a complaint. When management
learned that Carl had gotten involved they fired him, and Carl then decides to sue the
company for retaliation. He retained Danielle as his lawyer. Carl turns over to Danielle
the video that Pamela had given to him. The case received a lot of publicity and Danielle
gave some interviews about it on local TV. In one of those she showed Pamela’s video.
Pamela was very humiliated and emotionally upset when the video aired, and she has
sued Danielle for negligent infliction of emotional distress.
o Is she likely to win? Why or why not?
o The court held that there was no cause of action because there was no direct
relationship between Danielle the lawyer and Pamela the plaintiff
28
o This is a business relationship case, however here Danielle was Carls lawyer not
for Pamala
o No business relationship
o If no physical injury, then it has got to be a relationship case

Module 2C: Negligence- Breach of Duty


 Plaintiff must demonstrate exactly what the defendant did wrong and reasons why it is
wrong
 Explain why this conduct falls short of the standard
 You need fact plus reason in order to make a breach showing
 Plaintiff will claim this is unreasonable because…Give a reason- alcohol impairs your
ability to drive
 A behaviour might be short of the standard of reasonable prudence is that its costs
outweighs its benefits

Res Ipsa Loquitur


o Res Ipsa Loquitur is a doctrine used by plaintiffs who lack information about what the
defendant did wrong
o If you have not specified a particular breach, then how do you know you sued the right
person?
o Must establish that the likely hood that you sued the right person by showing that the
defendant had exclusive control over the injury causing item at the relevant time
 Byrne V Boadie 1863 Case from England. Mr. Byrne (Plaintiff) testified that he
was walking along Scotland Road when he passes a bakery and then evidently lost
consciousness by a barrel falling and hitting him on the head. As Mr. Byrne lost
consciousness he did not know who or how it hit him but he did sustain injuries.
Mr. Byrne brought a negligence claim against Mr. Boadle (Defendant) the owner of
the Bakery shop. The defendant’s shop was adjacent to the road on which Plaintiff
was walking, and the barrel appeared to have fallen, or was dropped from the shop.
In a regular case this would have been dismissed because no evident of a breach as
no indication to the breach.
 A plaintiff must persuade a jury that more likely than not the harm-causing event
does not occur in the absence of negligence.
 As the barrel was in exclusive control of the bakery then whatever screw up
occurred it is on them
o Res Ipsa Loquitur is a way that you can avoid a directed verdict by making 2
substitutes showing instead of direct evidence of a breach

2 Substitute Showings Instead of Direct Evidence of a Breach


1. You have to show that the accident is of a type/category which is normally associated
with negligence
2. You have to show that an accident of this type would normally do to the negligence of
someone in the defendant possession

 When you show these 2 things you get to the jury, as it is enough to satisfy the
prima facie case of breach.
 The jury can still come up with a verdict either way as there is no guarantee of a
victory just because you made a claim using Res Ipsa
29
Module 3A: Negligence- Causation- Factual & Proximate Causation
 On an essay question always do factual causation first(alphabetical F comes before P)
 The jury decides causation
 When multiple defendants or merged causes don’t use But for Test but substantial test

Factual Causation
This element is the point in the case where the plaintiff establishes a linkage or
o
connection between the breach and the injury suffered
 How do you show the connection/link between the 2?
o But for Test- (Latin for but for- sine qua non)
o Whether the breach was essential/necessary in producing the injury

 How does a defendant counter argue the But for Test?


 The argument usually starts like this:
o Even if I have been careful, you still would have gotten hurt

Multiple Defendants (2 merged causes)


 A classic example is 2 negligently set fires
 Example 1: Dave is smoking and throws his fag out the window, this is
not a reasonably prudent thing to do. The fag lands in a bed
of dry leaves and sets a fire (fire A). At the same time but a
mile away, Doug is camping in the woods and packs to go
home but forgets to extinguish his camp fire (fire B). The
wind picks up and the two fires join/merge and burn down
Pat house (plaintiff). Pat is now suing both Doug and Dave.
 In this kind of case we have a merged cause and the proper test in causation would be
the Substantial Factor Test

Substantial Factor Test (merged causes)


 In a substantial factor test analysis, you ask whether each breach contributed to the injury
in a significant or substantial way
 If a breach, had it been the only breach in the story would it have been capable of causing
all the harm by itself then it is defiantly a substantial factor.
 If a breach had 2 substantial factors, you hold both of them jointly in the fire hypo
(example1) either fire in existence would have caused the plaintiffs house to burn,
therefore both the fag tossing(fire-A) and leaving the camp fire on (fire-B)
carelessness, both would be treated as substantial factors. Both breaches should be
therefore considered factual causes. If you hold both of them being substantial
factors you hold both of them jointly liable

Alternative Causes Approach Test: Multiple Defendants


 Multiple defendants and only 1 true cause of harm but unascertainable cause case
 The plaintiff must establish his case by a preponderance (quality of facts) on every single
element
 Shifts the burden of proof to defendants

30
 Each defendant must show their negligence is not the actual cause “it was not me
because...”
 If neither can prove its not them, then you hold them both jointly liable

 Summers V Tice 1948 case- 3 guys(Moe, Larry, Curly) went hunting with shot guns
for quail. Quail are ground birds that you have to get
them out of their hiding by getting a stick and shaking
the bush and they fly out into the sky (beating the
bushes). When they fly out you shoot them with 1 pellet
and kill it you don’t kill it with a bunch of pellets
because it will vaporize the bird. As the birds are flying
in the air both Larry and Curly shoot at the same time but
missing the birds. However, 1 pellet hit Moe in the eye
leaving him blind in that eye. Moe then sues both Larry
and Curly for negligence. The issue here is that only 1
pellet blinded him, so only 1-person carelessness caused
his injury and the others did nothing, but the plaintiff
does not know which one. The courts shifted the burden
of proof so the defendants can talk their way out of the
case by proving that neither of them shot him.

Sample MBE Question on Multiple Defendants


 Pete was in a car driving on a country road and there were 2 farms on either side of the road.
One farm was owned by farmer Dave and the other was owned by famer Dough. Each
farmer owned a chestnut brown horse. Farmer Dave negligently let the fences that separated
his pasture from the road fallen in disrepair. Farmer Dough negligently left the gate open
that separated his pasture from the road. As Pete was driving on the country road he sees a
horse, to avoid hitting the horse he swerve out of the way and hits a tree. The horse gallops
away. Pete sue both Doug and Dave for their Negligence because he doesn’t know which
one actually caused it.
 There was only 1 horse on the road,
 Only 1 breach that could have caused the harm, but we don’t know which one,
 It is unascertainable
 It is up to each defendant to talk there way out of it if they can

Proximate Causation (fairness in liability)


 The plaintiff must convince that liability would be fair
 Proximate cause (fairness) will be judged based on whether what happened was a
foreseeable consequence of this particular breach
 It is fair for people to pay for their foreseeable consequence, but not fair to pay for
unforeseeable consequence
 First question you ask is if it is an unusual, freakish or bizarre case, then look at the
breach, catalogue the risk that you would have expected before hand and then check the
risk that materialized. That is a foreseeability analysis and you do that in the spirit of
fairness
 If fairness is self-evident/obvious you do not need to demonstrate

31
 Example1: Paul was waiting at the tube station to ask a TFL worker
directions. The worker was helping a Dan get on the tube and
shoved him in. Dan as he was getting shoved in dropped a
package and it burned Paul. Paulo was a foreseeable victim,
however what had happened was that Paul got burned.
 When asking if someone is foreseeable you have to think of what risks can come out
of that breach or what is associated with the breach.
 The breach is shoving and knocking a package

 What is associated with the breach and what risks come from that breach?
o People can fall- if they fall they can get a bruise or a broken bone
o Knocking a package down can cause the content to break
o Knocking a package down can cause the package to fall on someone’s foot and
cause an injury in a form of a broken foot
 These are all foreseeable risks BUT
 Shoving and knocking a box do not foreseeably cause explosions and fires.
 That was a big surprise and because it was a big surprise TFL won’t be liable.
 It would be unfair to make TFL liable because it played out in such a
surprising way, they couldn’t have known the box would burn someone.
 This was an unusual, freakish or bizarre case.

4 Precedent in Proximate Cause (important)


 Courts have determined that all these outcomes are foreseeable, liability is fair, that the
defendant should be treated as a proximate cause and the plaintiff wins.

Original plaintiff will always be liable in cases involving:

 1) Intervening Negligent Medical Treatment (mal practice)


 If the doctor makes a situation worse the original negligent defendant(he is the
proximate cause) will still be liable for all intervening negligent mal practice
cases that has happened after the injury that the defendant has caused because it
is foreseeable that a doctor sometimes might make a situation worse instead of
better. Some people will make errors in every profession, it is common but it
won’t happen all the time. It is not surprising so therefore it is foreseeable.
 That doctor will also be liable if he gets sued because he has not conformed to
the custom of other physicians

 2) Intervening Negligent Rescue


 Original defendant will still be liable if a rescuer injures the defendant more than
his original injuries.
 The courts have said that when you negligently hurt someone it is foreseeable
that rescuers will be attracted and sometimes they may make things worse rather
than better.
 It is foreseeable that the rescuers could make things worse, so the original
defendant will be liable and pay for it.

 3) Intervening Protection or Reaction Forces


32
 Original D will be liable if intervening act was caused by others trying to protect
themselves or a reaction of fear based on the original defendant’s negligent act
 Example 1: D runs a red light, enters an intersection crowded with
pedestrians. D hits a pedestrian and breaks his leg. The other
pedestrians start freaking out and in order to protect
themselves, in reaction they stampede to get out of the way and
one of these people steps in the face of the man with the
broken leg, who got hit by the car leaving the man with also a
disfigured face. Driver is liable for both the broken leg and the
disfigured face.

 4) Subsequent Disease or Accident


 Example 1: Defendant runs a red light, hits a pedestrian and breaks his
leg. Pedestrian gets a cast and crutches, however because
the pedestrian has never used crutches before he falls and
breaks his arm. The defendant will be liable for both
because it is foreseeable that if you leave someone in a
weekend condition they may have that second accident and
it’s only fair that D should pay.

Practice Question
Jenny and Pamela were having lunch at Diver Dan’s Seafood Restaurant. Jenny ordered a
shrimp cocktail appetizer. Diver Dan’s had failed to store the shrimp in a reasonable
fashion and it had become infected with bacteria. Shortly after finishing the shrimp Jenny
told Pamela she was feeling ill and excused herself to go to the bathroom. Sometime later
Pamela becomes concerned and decided to check on her. When entering the bathroom,
Pamela sees Jenny inside throwing up. Concentrating on Jenny, Pamela did not see a
puddle of vomit that Jenny had left on the tile floor of the bathroom just seconds earlier.
Pamela slipped in the puddle, fell and broke her arm.
 If Pamela sues Diver Dan’s for her broken arm, will she recover? Why or why not?
o Pamela did not have a valid claim because there was no proximate cause
o This is a freakish occurrence

 When we look at a freakish occurrence, we have to look at the risk


associated with the breach
 The breach was serving tainted food,

 What are the risks of serving tainted food?


o The risk is that people will get ill, but what happened here is a broken bone
o Broken bones do not normally result when serving people contaminated or rotten food
o This was an unforeseeable outcome and therefore the court felt that liability would be
unfair and exonerated the restaurant

Module 3B: Negligence- Damages


 Harm suffered
 In order to receive damages in Negligence plaintiff must have suffered a detriment
 The plaintiff must show actual harm or injury to complete the prima facie case

33
 The plaintiff can recover economic damages (medical expenses) and noneconomic
damages (pain and suffering)

Eggshell Skull Doctrine


 Under the Eggshell Skull Doctrine, once the elements have been established the plaintiff
will recover for all harm suffered, even if its surprisingly great in scope
 You take your plaintiff as you find your plaintiff
 If someone has a frailty of some kind, you are still liable
 Applies to every tort on exam, universal principle of tort law even to intentional torts

Module 3B: Negligence- Defence


 Historically we had a doctrine called contributory negligence, assumption of the risk
 These doctrines are minorities in the US and are only used in some jurisdictions
 The ones that are mostly used on the test is the rule of comparative negligence
 Comparative negligence comes in 2 schemes, they are pure comparative negligence and
partial comparative negligence
 Almost all states have rejected the rule that a plaintiff’s contributory negligence will
totally bar her recovery

Comparative Negligence
 The defendant must show that the plaintiff failed to exercise proper care of his own safety
 Proper care means reasonable prudence
 The amount of care that you should exercise to protect yourself from getting hurt is
exactly the same amount of care as you should exercise from preventing others from
getting hurt, the care of a reasonably prudent person
 Proper care can also include a self-protected statute
 Example of a self-protect statute: the law the forbids J-walking, this statute is to protect
you from your own foolish act because that would be a way for you to get hurt if you
don’t obey the law
 Plaintiffs damage recovery can be reduced based on their fault and there is no legal rule
that will assign the reduction of those numbers (%), it can be any %.
 The % is based on fault that has been assigned to them by the jury after they weigh the
negligence of the 2 parties, this is a damage reduction
 Example 1: Defendant offers evidence that plaintiff failed to exercise
proper care for their own safety. The jury will then be
instructed to weigh the carelessness of the 2 litigants and
to assign each litigant a percentage number. The plaintiff’s
recovery is reduced based on P’s percentage of fault that
has been assigned to them by the jury, this is a damage
reduction which is the consequence of the P’s fault and
there are no legal rules that set a certain %.
 Example 2: P J-walked and got hit by D a drunk driver. P sustained
$100,000worth of damages. The jury said that P the J-
walker was 20% at fault and D the drunk driver was 80%
at fault. Therefore, P will only recover $80,000 as the
damages were reduced by his own damages of fault.

34
Pure Comparative Negligence (default on the bar exam, unless they instruct you otherwise)
 In this system we go strictly by the numbers
 Even if the plaintiff is assigned the higher number of faults, the plaintiff still goes home
with some recovery even if she was primarily 90% at fault.

Partial/modified Comparative Negligence


 A plaintiff’s fault 50% or less reduces recovery
 A plaintiff’s fault over 50% is an absolute bar and the plaintiff gets $0, they are barred
from recovering anything

Module 4: Strict Liability


 There are 3 causes of action in strict liability
 Will be listed by importance
 Strict liability for animals
 Strict liability for abnormally dangerous activities
 Strict liability for Products

Strict Liability for Animals


 Animals cases requires you to distinguish between 2 broad groups of animals:

 1) Domesticated Animals
 Domesticated animals are house pets and livestock
 House pets are cats and dogs
 Livestock are agriculture animals such as cows, horses, pigs,
ducks, goats, sheep and chickens to produce labor and
commodities such as meat, eggs, milk, fur, leather
 You are not strictly liable for domesticated animals, not strict liability
 If someone gets hurt by any of these animals there is no strict liability and they will have
to establish some negligence on your part

 Exception: If your animal has viscous propensities and these are known to you, NOW you
will be strictly liable, it is strict liability for keeping a domesticated animal with
known viscous propensities. This viscous propensity has to be to this main
species not to the entire specie. However, animals that cause a viscous
propensity does not apply to trespassers on your land, it only applies to public
people on the street and to social guest at your home or business customers at
your place of business.
 All mules of donkeys will kick occasionally, that his behaviour common to
a mule and no viscous propensities
 All bulls will put their heads down and charge occasionally, that does not
mean it has viscous propensities
 A dog that bites humans IS a viscous propensity because dogs have been
breaded to not do that, but some do. You could have known if once your
dog bites someone, once it does you know that your dog is the rare dog
that bites people and if you continue to keep that dog you will be strictly
liable if he bites again.

35
o The first bite puts you on notice, second bite strict liability

 2) Wild Animals
 If dealing with wild animals, then there is strict liability
 No knowledge or first bite needed
 Safety precautions are irrelevant because its strict liability
 Wild animals- elephants, monkeys, lions, tigers, bears

Strict Liability for Abnormally Dangerous Activities


 Abnormally dangerous activities is an activity that meets a 2-part test:

 1) Activity creates a foreseeable risk of serious harm, even when reasonable


care is exercised
 Reasonable care cannot shrink the amount of risk down to a comfortable
level, the amount of risk will still strict liability no matter what.
 The activity is abnormally dangerous, even when engaged in carefully it
still produces a risk of a bad outcome
 2) Activity is uncommon in the community where it is being conducted
 If everyone is doing it, it might be dangerous but by definition it’s not
abnormally dangerous because everyone is doing it so its normal

There are 3 Types of Abnormally Dangerous Activities:


 Anything involving:

 1) Explosives
 Dynamite, PNT(explosive material), nitrogen, plastic(soft & hand-mouldable sold
form of explosive material), nitroglycerin or any explosive materials

 2) Highly Toxic Chemical or Biological Material


 Biological material is usually blood, urine, human tissue, organs, cells,
tissues, living and dead people. Some can pose a threat such as virus or
toxins from a biological source that is affected and can affect/threat to
human health.
 Example 1: In a lab they are trying to find a cure for
the Ebola vaccine. In the lab they have
live Ebola cultures, that is an abnormally
dangerous activity

 3) Radiation or Nuclear Energy


 Safety precautions are irrelevant(it doesn’t matter if they are safe), it is always strict
liability
 Nuclear power plants emit radiation
 Demolition companies use radiation

Module 4: Strict Liability for Products


 In a Strict Liability case the plaintiff must satisfy the 4 elements
 Strict liability refers to both industrial and consumer products
36
 If someone gets hurt by a product they will have multiple possible causes of actions and
possibly multiple defendants such as a wholesaler, manufacturer and retailer
 There is not requirement of privity of contract to bring a strict liability claim, which
means you can sue people that you did not deal with directly

 1) Consumer Products
 It can be a small appliance you have in your home such as:
 A toaster, blender, waffle iron or it can be a car, boat, motorcycle
 It can be a food product such as a can of tuna fish, bottle of olives

 2) Industrial Products
 It can also be something that you encounter in a factory:
 A forklift truck, drill press or industrial machinery that lack safety
features such as guards to prevent your hand from getting mangled

 Example 1: Paul has a blender at home and it hurts him


 Paul may be able to bring a negligence claim against the company that manufactured
the blender.
 Maybe the manufacturer put it together and did something careless such as
forgetting to tightening some screws.
 If Paul does not have direct evidence of what they did wrong he may be able to rely
on Res ipsa loquitor.
 Paul may have an action against them for the breach of warranty of merchantability
that rises under Article 2 of the UCC
 If they lied about the product, Paul may have a claim for misrepresentation or fraud
 Paul may have a claim for strict liability
 There is many different situations that could be tested, make sure YOU analyze the
under the right certain theory that the examiners asks about

If the call of the questions says:


 Example 2: Patricia sued the department store for negligence
 If you use the strict liability analysis you will get it wrong
 If it is strict liability it has to satisfy the elements

There are 4 Elements that a Plaintiff Must Show in Strict Liability


 In strict liability the plaintiff must show that:
1. The Defendant is a Merchant
2. The Product Must be Defective
3. The Product has Not Been Altered Since It Left the Defendants Hands
4. The

 1) The Defendant is a Merchant


 You can’t rely on strict liability unless the person you are suing is a merchant
 A merchant is someone who routinely deals in goods of this time

4 Common Mistakes:

37
 1) A Casual Seller is not a merchant and cannot be strictly liable. A casual seller
is you or me selling used goods online or in the newspaper
 If you sell your car online to someone and the breaks fail you may be
liable in another tort but not in strict liability

 2) Service Providers sometimes make products available incidental to the


service, but they are not considered merchants of those products, therefore they
are not strictly liable if something goes wrong with the product
 You go to the doctor’s office and sit on a chair and it breaks, the doctor
is not strictly liable because the doctor is not a merchant of chairs, he
is a vendor of professional medical services. You are an invitee and
may have a premises liability negligence claim but not strict liability

 3) Commercial Lessors are people who are in the business leasing stuff. These
people ARE MERCHANTS and can be strictly liable.
 A rental car company. You rent a car to go to Florida and get 5 miles
away when the breaks fail on that rental car.
 The rental car is strictly liable

 4) Parties up the Distribution Chain with whom you did not deal with directly
are also merchants and are strictly liable
 Wholesaler
 Manufacturer
 Retailer

 2) The Product Must be Defective


 The plaintiff must show that the product suffers from a defect
 On an essay, you must analysis the 3 different kinds of defective any one will
allow the plaintiff to move on with the case

3 Types of Defects, in which only 1 needs to be satisfied:

 1. A Manufacturing Defect: (1 in a million)


 A product has a manufacturing defect when it differs from all the others that came off
the same assembly line, in a way that makes it more dangerous then consumers would
expect
 A product has a manufacturing defect when it departs from its intended design that
makes it defective
 If all the other products has 4 bolts and this one has 3 bolts
 If all the other products have a smooth edge and this one has a jagged
edge, it can be a defective product
 In a manufacturing defect, safety precautions do not matter because its strict
liability. Any safety methods they used does not matter, its strict liability.

 2. A Design Defect:
 The law has been extremely controversial, political and unsettled for a long period of time

38
 Products have a design defect, if costs of the design outweigh the benefits of the design,
such that a reasonable merchant would not have put it on the market for sale
 Cost of the design are usually its tenancy to hurt you
 Benefits of a design can be things like, it makes the product easier to use, or more
comfortable to hold, quicker to use
 Benefits can be things that make the cost of the item lower, they reduce the
manufacturing expenses
 There is always a balance between incorporating safety features that may make the
product heavy or cumbersome or slow or expensive or leaving those safety features off in
order to make the product light and easy to use, but more risky
 So, the test is due to the cost of danger that outweighs the benefit types listed above such
that a reasonable person wouldn’t market the product
 The law of design defect is a strict liability claim, however it is a form of negligence
liability, so you do use reasonable person
 How does the plaintiff establish that the cost outweighs the benefits(more safety in the design)?
o It is by offering evidence of a reasonable alternative design, that passes the 3-part test
o They could have built it better by not making anything negative happen and their failure
to do this is evidence of what they put on the market is unreasonable
3 Part Alternative Design Test:

 1) Its Safer Than the Product Marketed


 They hypothetical alternative design is a safer design

 2) It is Economically Feasible
 The alternative design is about the same price to manufacture
 It has to be cost effective

 3) Its Practical
 It doesn’t make the product difficult to use

 If an alternative design exists then that suggests that the cost of the actual design
outweighs its benefit because there is a better way to build it and that makes the
product defective
 Example 1: In the 80’s the design of baby cribs had vertical slats in
where they were more widely spaced apart then today.
Some babies would be able to stick their heads in the slats,
get trapped, choke and a tragedy would result. This has
happened to Patricia and so she brought a cause of action
that claimed that the crib was defectively designed. It had
a design defect and the costs (the risks) of that product
outweighed its benefits. Pat would show that by saying “I
can show you an alternative way of designing that crib”.
The alternative was it moving the slats closer together, so
that the baby can’t stick its head in-between the slats.
 Does that alternative design meet the 3-part test?
o 1. Is it safer? Yes, no straggled babies
o 2. Is it cost effective/economically feasible? Yes, spend a few more dollars for 1
piece of wood, then just reposition
39
o 3. Is it practical? Yes, the crib still functions perfectly as a crib. Baby still gets air,
light, visual stimulation, you can still see the baby, touch the
baby and put your hand through the slats to touch the baby
 Therefore, it is a good alternative design and those old cribs were defective

 3. An Information Defect:
 If a product has residual risks that cannot be designed out and if that risk is not
obvious or apparent to the users/consumers, the product is defective if it lacks
adequate warnings and instructions
 If the risk is hidden the product needs a warning and the warning must be
adequate
 Not all warnings are created equal
o Every product has some residual/remaining risk that can’t be designed away
such as a knife. A knife can cut and hurt someone, if you make it dull it will
not function as a knife and that’s not practical.
o Cutting yourself is an obvious risk, so you don’t need a label that says
“warning do not run tongue around blade” that is unnecessary

Issues with Adequate Warnings:


 When looking at the warning, ask yourself if the warning is adequate or if it would be
easily improved upon

 1. Placement
 Putting a warning on page 8 of the instructions might not be adequate
 Adequate should be on a big red sticker on the point of use

 2. Comprehensibility
 If the wording is to complicated or in English only as it should be bilingual
 Perhaps the warning should not only use wording but also icons for people who
don’t know how to read

 3. Risk Mitigation Information


 Sometimes warnings are not enough unless it is accompanied by risk mitigation
 The warning should show you how to mitigate the risk if there a logical way to
mitigate the risk.
o Like when using a pesticide, the warning label says “warning toxic fumes”
and then it should say “wear mask while using”(mitigating the risk)

 3) The Plaintiff Must Show That the Product has Not Been Altered
Since It Left the Defendants/Manufacturers Hands
 There is a presumption that the product has not been altered, so long as it
moved in ordinary channels of distribution
 As in you got it from an ordinary retailer, who got it from the wholesaler, who
got it from the manufacturer
 There is a presumption that the product that you buy from the retailer is in the
same shape as when it left the factory floor, as it moved in ordinary channels of
distribution

40

The plaintiff can rely on the presumption to satisfy the element and then it’s up
to the defendant to offer contrary evidence, if any
 This presumption does not apply to items purchased used, as it could have been
abused/misused by the previous owner in many different ways
 If a used product hurts you, it is on you to show that it is in the same condition
that is was when it left the manufacturer hands, if you go to sue the
manufacturer

 4) The Plaintiff Must Show That at the Time of Injury he was making a
Foreseeable Use of the Product
 Plaintiff has to be making a foreseeable use of the product at the time of injury
 A foreseeable use is not necessarily an appropriate or proper use
 Many missuses, many inappropriate uses are nonetheless entirely foreseeable
 Do not think the test is if the product is being misused, the test is whether the use
is foreseeable
 The test it foreseeable use not proper use
 In foreseeable use, there are people that are wacky that can use products in
unforeseeable ways like if you get a lawn mower and try to cut your hair with it
that is an unforeseeable use and will be no liability for it. (jackass movie)
 Example 1: Pete needs something in a high shelf in his home, to get it he
stands up on a chair and then the chair collapses because it
can’t support his weight. He brings a claim against the
chair manufacturer. It is a manufacturing defect, it is the 1
in a million bad chair that can’t support his weight. The
manufacturing company says that he was not using the chair
in the correct manner as chairs should not be used as step
ladders, they should be used as seating only. The courts said
that’s irrelevant. The question is whether it is foreseeable
that people will stand up on chairs. The answer on
foreseeable is, yes as everyone does it.

Strict Liability Defense of Comparative Responsibility:


 All 3 of the Strict Liability Torts only have 1 concept of an available defence
 The defence is similary to Comparative Negligence but it is a different name
 The defence is called Comparative Responsibility
 If the defendant can prove that the plaintiff in any strict liability case was careless,
was foolish, was arrogant, was cocky and did not take proper effort to protect
his/her own safety and confronted a known risk then:
 The jury will assign some percentage of responsibility to the plaintiff and the
plaintiff’s recovery will be reduced
 Example 1: Your neighbour has a tiger and you go with a stick
and poke it through the fence. The tiger gets
annoyed and attacks you. You try to sue the
neighbour for strict liability for keeping a tiger, but
you will be charged with a very high percentage of
the fault because you were taunting the animal
41
 Example 2: Your toaster was sparking earlier but you were very
keen on having a piece of toast so you use it
anyways. The toaster then explodes and you try to
sue the manufacturer for strict liability for a
manufacturing defect, it was that 1 in a million bad
toaster. You will be charged with a very high
percentage of fault because you knew it was
sparking so why would you use it.

Practice Question:
 Delta makes toasters which it sells to retailers for distribution to consumers. Pamela bought
a Delta brand toaster from the SuperDeal electronics store, and when she got home and
plugged it in it gave her a severe shock. She sued Delta, the manufacturer, in strict liability.
At trial, she offers evidence of her injury. Delta offers evidence that the specific toaster
Pamala purchased had been knocked off of a high shelf by an employee at the electronic
store, who placed it back on the shelf. Delta also offers evidence that an examination of the
toaster showed that some safety circuits were disconnected, and an expert testified that this
was likely due to the result of the product having been dropped from a significant height.
 If all this evidence is admitted at trial and the case is sent to the jury, will Pamela be able
to win her strict liability claim against Delta? Why or Why not?
o Pamela will not succeed as this product has been altered since it left Deltas control
o Normally she would be able to rely on the presumption that it has not been altered,
but here the defendant rebutted the presumption with extensive evidence that it had
been dropped by the retailer and that dropping by the retailer caused damage and that
damage was directly traceable to the nature of the injury because it involved the
safety circuits
o That evidence rebuts the presumption and in effect because this defect was
introduced after it left their hands, they are not responsible

 If we held Delta liable, with the facts of this question we would have turned Delta into an
insurance company.
 Delta is strictly liable, but they are only strictly liable for things that happened on their
watch. This toaster was altered and that’s why they are off the hook.

Module 5: Nuisance (maybe 1 question not heavily tested)


 A Nuisance is a tort that involves real estate/property
 Nuisance is defined as unreasonable interference with plaintiff’s ability to use and enjoy
property
 Unreasonable does not mean that the defendant is necessarily acting unreasonably, what
it means is that the amount of interference is unreasonable
 It means that the defendant is doing something that makes a nearby land owner
miserable, D has rendered P’s condition intolerable
 Many but not all nuisances’ cases involve inconsistent land use, just the unfortunate jacks
the position of 2 usually commercial enterprises, sometimes a business and a residence
that should have never have been near each other
 A common example is a smoking factory next to a sanitarium for people suffering
for asthma, these 2 cannot be sole locating because of the smoke coming from the

42
factory would make it difficult to run that sanitarium with the people with asthma,
as it would make them worse.
 Example 1: A recording studio for a heavy metal band located next to a
recovery facility for people who had nervous break downs
 Example 2: A plant nursey that sold cow manure in bulk as fertilizer
next door to a French restaurant with an outdoor cafe
 Theses 2 business should not be next door to each other and would cause a nuisance to
one another
 A Nuisance can also involve spite or inconsideration
 If you deliberately shine lights onto your neighbour’s property, your neighbour
may have a cause of action against you for nuisance. You are being malicious and
your neighbour can never go to sleep because the lights are so bright that even
with drapes or curtains the neighbour’s house is still lite up
 Your neighbour is inconsiderate and throws loud parties of all hours of the night
or day, that can also be a nuisance
 Your neighbour is running an illicit operation, a drug supermarket next door.
There are strange people coming and going, cars parked on the street, weird
noises and occasional sounds of gun shots. This could also be a nuisance

 The key question is a balancing of the interests


 The defendant is entitled to use his land as he wishes, if it’s a lawful purpose. If you
want to have a recording studio for a heavy metal band and it’s not in any ordinance,
then you are entitled to do that BUT
 Your neighbour should also be free from misery, your neighbour should not be
subjected to such a position that his property becomes worthless and has to vacate it
 If you’re going to make the parcel next door uninhabitably, then just buy the parcel
next door because you can’t do this as it is considered a nuisance
 We are all obligated to put up with minor inconveniences, those don’t rise to the level
of a nuisance
 Your neighbour likes to mown the lawn 8am Sunday mornings and you like to
sleep in longer then 8am Sunday mornings, that’s too bad because it is not
enough to rise to a level of a nuisance
 If you live across the street from a church and you hear church bells. Church
bells annoy you because you work from home and it disturbs your
concentration, this is still not enough to rise to a level of nuisance
 A level of nuisance has to be something that makes your life intolerable

 What do you do if you get a Nuisance Question on the Multistate?


o You look for the choice that imbodies the legal rule
o Look for a choice that says:
o “plaintiff will win if defendants conduct interfered with the ability to use and enjoy
the property to an unreasonable degree” or
o “ Plaintiff will win if the balance of interest tips to the plaintiff’s favour “

43
Module 6: Miscellaneous Topics Including Vicarious Liability
 Topics and categories go in order as most important and will likely be on the test

Vicarious Liability
 It’s possible to hold the direct tortfeasor liable but also a secondary party liable as well
 The secondary party will usually be liable because of a relationship
 Vicarious liability is predicated on 4 different relationships

4 Categories of Relationships in Vicarious Liability:

 1. Employer  Employee
 Under a vicarious liability theory, if an employee goes out and commits a tort,
the employer will be liable as well; if the tort was committed “within the scope
of employment”
 What is the scope of employment?

 Is an Intentional Tort Within the Scope of Employment?


 Example 1: Diane runs a flower business and has an employee delivery
driver named. Diane’s employee Dave runs a red light while
delivering flowers, Diane as the employer is vicariously
liable for her employee Dave running the red light.

 Example 2: A supermarket has Don as an employee, as he is stalking


cans on the shelf. A customer says to Don the employee
“excuse me where to you keep the canned pasta?” Don then
turns around and kicks the customer in the growing’s.
 Is the Supermarket Liable for Dons Intentional Tort?
o No, intentional torts are outside the scope of employment. BUT it’s not an absolute rule,
there is circumstances where these intentional torts come back into the scope of
employment such as these situations:

Intentional Torts that are in the Scope of Employment

 1. If the Job Involves a Use of Force, Then Misusing the Force is Within the
Scope of Employment
 Jobs that involve use of force are store security guards, night club
bouncers
 It gives the nightclub the incentive to hire calm people because if
you are going to allow them to use force we rather not you hire
them, so we will make you liable for any tort they have
committed
o A night club bouncer loses his cool as he see a romantic
rival so he starts wailing on him and starts punching
him, committing a battery. The nightclub will be
vicarious liable for the bouncer’s battery.

 2. If the Job Leads to Friction or Tension


44
 It also gives creates in incentive to hire calm minded people for that kinds of
work because if you are going to allow them to use force we rather you not hire
them, so we will make you liable for any tort they commit
o The guy who repossesses your automobile if you don’t make payments, if
that guy loses his cool and punches a customer. The bank or the finance
company that employed him, they are going to be vicariously liable

 3. Any Intentional Tort That is Done to Serve the Bosses Purpose, will be Within
the Scope of Employment
 The store security guard, who starts randomly detaining and searching
customers to cut down on shop lifting, is committing false imprisonment.
 That store will be vicariously liable because that was done to serve the
stores purpose

 2. Hiring Party  Independent Contractor


 The general rule is no liability in torts for the independent contractor, but there is an
exception
 If you hire an independent contractor to work on your business premises, you
will be vicariously liable if that independent contractor hurts a customer or an
invitee while at work, working for you.
 You owe a non-delegable duty of care to your customers, you remain
responsible to them
 Example 1: You hire painters to paint your convince store, they
have to work while the store is open because the store
is open 24/7. On route to the store they run a red light
and hit a pedestrian, you are not liable. The painters are
independent contractors they are not your employees,
so no vicarious liability. However, once they get to the
store they set up a ladder and put a tub of paint on the
shelf and then accidently knocks it over and hits a
customer in the head. You will be vicariously liable for
that.

 3. Automobile Owner  Automobile Driver


 An owner of a car is not vicariously liable for the torts of a driver
 If I lend you my car, I am not responsible if you go out and get drunk and
drive drunk, or if you speed or if you run a red light
 The owner of the car will be vicariously liable IF I ask you to take my car to
do an errand for me because we are in a different relationship, which is
Principle  Agent
o You are always vicariously liable for the torts of your agents

 4. Parent  Child
 No vicarious liability at all, parents are not liable for the torts of their kids
 If you get hurt by a kid, you sue the kid. Kids are covered by insurance and
sometimes have their own assets, but the parents are no vicariously liable.
Multiple Defendant Issues:
 Comparative Contribution Rule- where the defendant gets a % of their money back
45
 Indemnification- where the defendant gets the full reimbursement
 Non-manufacturer- In a strict liability claim a wholesaler and retailer can be fully
indemnified by the manufacturer

 Example 1: Assume that the plaintiff has sued multiple defendants


Ali, Bill and Carl. The plaintiff will have won the claim
and recovered all the money from 1 of the defendants. It is
a $100,000 judgement and the plaintiff got it all from Ali.

 Does Ali have any rights against his co-defendants Bill and Carl to get some of that
money back? Is there a right of reimbursement?
o Yes, Contribution is available and the amount is set by percentages signed by the
jury
o The jury gives each defendant a percentage and we just follow those numbers

 If the jury said:


 Ali was 10% at fault and
 Bill was 30% at fault and
 Carl was 60% at fault
 Then Ali gets $30,000 back from Bill and
 $60,000 back from Carl
 This will happen under the Comparative Contribution Rule

There are 2 Situations Where Defendants Can Get all their Money Back
 Full reimbursement and not only a percentage

 1) Indemnification:
 It arises in a situation where the defendant is vicariously liable and the
defendant paid the plaintiff.
 A vicariously liable party can get full reimbursement or indemnity from the
active tortfeasor

 2) A Non-Manufacturer
 Typically, a wholesaler or retailer can get full indemnification from a
manufacturer in a strict liability claim
 Example 1: If P sues Home Depot and Black & Decker because
P bought a defective power tool and P wins against
both of them. P recovers all the money from Home
Depot. Home Depot can then get an indemnity from
Black & Decker. Home Depot goes to Black & Deck
and says “ I just paid the plaintiff now give me that
money back”. That has the effect of shifting the loss
up the chain so that it rests on the shoulders of the
manufacturer.
Loss of Consortium :

46
Whenever a married person is injured and has a valid tort claim, their spouse also gets a
separate independent claim
 The uninjured spouse gets a second separate independent claim
 The claim is derivative(it is based on another source), that means any defences that can
be asserted against the plaintiffs injured spouse can also be asserted against the
uninjured consortium spouse
 If there is an argument of consent, then also raise that against the
consortium party
 If there is an argument of comparative negligence, then also raise that
against the consortium party
 Why is There a Loss of Consortium Claim? Why Give the Spouse a Remedy?
o It is to allow recovery of 3 types of damages that would otherwise be uncompensated

3 Types of Damages That Would Otherwise Be Uncompensated are:


 The loss of consortium claim (spouse) can recover for:

 1. Loss of Household Services


 The argument is that with a spouse injured there is no one to do the
dishes, or laundry, or cleaning, or cooking, or taking the kids to school
and they might need to hire someone to do these things and get money
for it.

 2. Loss of Society
 Loss of society is loss of companionship
 The argument is that with my spouse injured the other spouse has no one
to talk to and you do get money for that.

 3. Loss of Sex
 The argument is with my spouse injured, I can’t have sex with them. You
also get money for this.

NY Bar - Video 3- Tort Law


 Module 7: Defamation and Invasion of Privacy

A Common Law Action of Defamation has 3 Elements:

 1) The plaintiff must show that the defendant made a defamatory statement that
specifically identifies the plaintiff
 A defamatory statement is defined as a statement which tends to adversely affect
your reputation

47
 The law viewed your reputation as an economically valuable assets(hire you, dal
with you services.

 2) It is not enough that the statement be maid, there also must be a publication of the
statement

 1. Defamatory statement-a statement that lowers your reputation in the eyes of the community:
 It involves in allegation of fact that negatively affects a specific character trait
 A statement that has a true/false analysis
 A statement of opinion is not defamatory because it is not subject to a true/false
analysis unless it applies fact
 Name calling is not defamatory
 You do not need to state the person’s name
 The plaintiff must be alive at the time the statement is made

 2. Publication is satisfied if I share the statement with 1 other person


 The more people D shares the statement with the more harm to the reputation and
more damage award when you sue
 If it is said to only the plaintiff in private and is not published it is not a defamation
 Publication doesn’t have to be intentional, negligent publication is sufficient as
leaving a school PA on and everyone hearing
 Anyone who repeats the publication are liable as re publishers

Damages in Defamation
 Damage will be presumed if the defamation is:

 Libel (in writing or other permanent form) or

 Slander (spoken)
 within one of the four per se categories (business or profession, loathsome
disease, serious crime, or unchastity of a woman (sex before marriage)

 Otherwise Special (pecuniary) damages must be shown if not slander per se


 A statement that john is cheating on his wife, or a man is having sex before
marriage
 Must show economic harm (money and work)
 Hurt feelings do not count
Practice Question
 2 women, Debbie and Jane, both parents of young children, were talking outside of a
school one afternoon while they were waiting for their children. Jane asked Debbie for
the phone number of the woman she employed in her home to take care of her children.
Debbie, knowing that a reliable childcare provider like the woman she has is hard to find,
did not want to share the information with Jane because she was afraid Jane would hire
the woman away. She instead said, “you don’t want her number, we discovered that she
was drunk last week when we came home from work.” This accusation was false but it
quickly spread throughout the neighborhood. Pamela, the woman employed by Debbie,
learns of the accusation. She sues Debbie for defamation and establishes the above facts.

48
 Which of the following additional facts must Pamela show in order to succeed?
A. That she suffered special damages by reason of Debbie’s statement
B. That she lost other childcare jobs because of the statement
C. That the statement was extreme and outrageous because any drinking of alcohol is
against Pamela’s personal moral beliefs
D. No additional facts - (this is slander per se so damages are presumed)

There are 3 Affirmative Defense:


 The burden of proof will be with the defendant

 1) Consent- It can be expressed or implied

 2) Truth – D can show that the statement is factually accurate and that defeats the claim

 3) Privilege- There are 2 kinds of Privilege:

1) Absolute Privilege- is based on the identity of the defendant.


 The communication between spouses
 Officers of the 3 branches of government, in the conduct of their official work

2) Qualified Privilege- arises only when there is a public interest encouraging candor
 References and recommendations- Person invoking the privilege must have
reasonable and good faith basis for making the statement
 Honest mistake when
 You must confine yourself to matters that are relevant to the subject at hand,
you can’t go off and tangents
 As long as you speak in good faith, you will be protected by the qualified privilege

If the Subject of the Defendants Statement is a Matter of Public Concern


 Something that is news worthy
 Wide spread public interest
 Public discourse
 Honest integrity of public officials or public concern
 Concerning celebrities or sports figures

Apply Constitutional Rules if the Plaintiff is a Public Figure


 The burden of proof will be with the plaintiff

 1) The plaintiff must prove that the statement was false

 2) Public officials/ figures must prove “actual malice,” i.e., that the statement was
made with knowledge/fault of its falsity or reckless disregard of its truth or falsity

 New York Times V Sullivan- All plaintiffs should be obligated to prove falsity,
fault usually negligence
The Common Law of Privacy

49
1) Appropriation of the plaintiff’s picture or name
 1.Unauthorized use of the plaintiff’s picture or name for the defendant’s
commercial advantage

 2. Limited to the advertisement or promotion of products or services


 You can take a celebrity’s photo and put it on the newspaper but not on a poster

2) Intrusion on the plaintiff’s affairs or seclusion


 1. An act of prying or intruding on the plaintiff’s private affairs or seclusion that
would be highly offensive to a reasonable person
 Wiretapping, ease dropping, hacking emails, secrete cameras in your home
 You must be in a place where you have a reasonable expectation of privacy
 Public spaces you don’t have a reasonable expectation of privacy

3) Publication of facts placing the plaintiff in a false light


 1. The publication of facts about the plaintiff putting her in a false light in the public
eye in a way that would be highly offensive to a reasonable person
 Going around town telling people lies about the plaintiff and it characterizes
him as a false light in the public eye
 Can be both defamation (economic damages) and false light (emotional and
dignitary damages)
 2. Actual malice must be shown if the publication is in the public interest

4) Public disclosure of private facts about the plaintiff


 1. The public disclosure of private information about the plaintiff such that the
disclosure would be highly offensive to a reasonable person

 2. Public disclosure requires publicity, not just publication to a few people


 Celebrity is exceptions

 Defenses:
 Consent is a defense to all 4 claims
 Absolute or Qualified Privileges- is a defense to only false light and disclosure claims

50

You might also like